Difference between revisions of "2016 AMC 10A Problems/Problem 17"

(Pattern Solution)
(Pattern Solution)
Line 41: Line 41:
  
 
Dividing both sides by <math>-1</math>, we derive <math>\frac{\frac{N}{5}-1}{N+1}</math> <math>></math> <math>\frac{79}{400}</math>. (Switch the inequality sign when dividing by <math>-1</math>)
 
Dividing both sides by <math>-1</math>, we derive <math>\frac{\frac{N}{5}-1}{N+1}</math> <math>></math> <math>\frac{79}{400}</math>. (Switch the inequality sign when dividing by <math>-1</math>)
 +
 +
We then cross multiply to get <math>80N - 400 > 79N + 79</math>
 +
 +
Finally we get <math>N > 479</math>
 +
 +
To get <math>N = 480</math>
 +
 +
So the sum of the digits of <math>N</math> = <math>\boxed{\textbf{(A) } 12}</math>.
  
 
==See Also==
 
==See Also==

Revision as of 12:52, 20 May 2016

Problem

Let $N$ be a positive multiple of $5$. One red ball and $N$ green balls are arranged in a line in random order. Let $P(N)$ be the probability that at least $\tfrac{3}{5}$ of the green balls are on the same side of the red ball. Observe that $P(5)=1$ and that $P(N)$ approaches $\tfrac{4}{5}$ as $N$ grows large. What is the sum of the digits of the least value of $N$ such that $P(N) < \tfrac{321}{400}$?

$\textbf{(A) } 12 \qquad \textbf{(B) } 14 \qquad \textbf{(C) }16 \qquad \textbf{(D) } 18 \qquad \textbf{(E) } 20$

Solution

Let $n = \frac{N}{5}$. Then, consider $5$ blocks of $n$ green balls in a line, along with the red ball. Shuffling the line is equivalent to choosing one of the $N + 1$ positions between the green balls to insert the red ball. Less than $\frac{3}{5}$ of the green balls will be on the same side of the red ball if the red ball is inserted in the middle block of $n$ balls, and there are $n - 1$ positions where this happens. Thus, $P(N) = 1 - \frac{n - 1}{N + 1} = \frac{4n + 2}{5n + 1}$, so

\[P(N) = \frac{4n + 2}{5n + 1} < \frac{321}{400}.\]

Multiplying both sides of the inequality by $400(5n+1)$, we have

\[400(4n+2)<321(5n+1),\]

and by the distributive property,

\[1600n+800<1605n+321.\]

Subtracting $1600n+321$ on both sides of the inequality gives us

\[479<5n.\]

Therefore, $N=5n>479$, so the least possible value of $N$ is $480$. The sum of the digits of $480$ is $\boxed{\textbf{(A) } 12}$.

Pattern Solution

Let $N$ $=$ $5$, $P(N)$ $=$ 1


Let $N$ $=$ $10$, $P(N)$ $=$ $\frac{10}{11}$


Let $N$ $=$ $15$, $P(N)$ $=$ $\frac{14}{16}$


Notice that the fraction can be written as $1$ $-$ $\frac{\frac{N}{5}-1}{N+1}$

Now it's quite simple to write the inequality as $1$ $-$ $\frac{\frac{N}{5}-1}{N+1}$ $<$ $\frac{321}{400}$

We can subtract $1$ on both sides to obtain $-$$\frac{\frac{N}{5}-1}{N+1}$ $<$ $-$$\frac{79}{400}$

Dividing both sides by $-1$, we derive $\frac{\frac{N}{5}-1}{N+1}$ $>$ $\frac{79}{400}$. (Switch the inequality sign when dividing by $-1$)

We then cross multiply to get $80N - 400 > 79N + 79$

Finally we get $N > 479$

To get $N = 480$

So the sum of the digits of $N$ = $\boxed{\textbf{(A) } 12}$.

See Also

2016 AMC 10A (ProblemsAnswer KeyResources)
Preceded by
Problem 16
Followed by
Problem 18
1 2 3 4 5 6 7 8 9 10 11 12 13 14 15 16 17 18 19 20 21 22 23 24 25
All AMC 10 Problems and Solutions
2016 AMC 12A (ProblemsAnswer KeyResources)
Preceded by
Problem 12
Followed by
Problem 14
1 2 3 4 5 6 7 8 9 10 11 12 13 14 15 16 17 18 19 20 21 22 23 24 25
All AMC 12 Problems and Solutions

The problems on this page are copyrighted by the Mathematical Association of America's American Mathematics Competitions. AMC logo.png